Fibonacci Sequence converge exercise

  • Thread starter Thread starter Calabi_Yau
  • Start date Start date
  • Tags Tags
    Exercise Sequence
Click For Summary
The discussion focuses on proving the convergence of the sequence un, defined as un = Fn+1/Fn, to the golden ratio φ. Participants explore the inequality |un - φ| ≤ (1/φ)|un-1 - φ|, leading to the conclusion that as n approaches infinity, un converges to φ. One user successfully demonstrates that as n-1 approaches infinity, the right side of the inequality approaches zero, confirming convergence. However, there is difficulty in transitioning from the first to the second inequality, particularly in relating un-1 to u1 and the term involving φn-1. The conversation concludes with a recognition of the pattern in the inequalities, facilitating the proof of convergence.
Calabi_Yau
Messages
35
Reaction score
1
Let Fn denote the Fibonacci sequence.
un is the sequence given by: un= Fn+1/Fn. Show that mod(un - \phi) \leq\frac{1}{\phi}mod(un-1-\phi) and therefore mod(un - \phi) \leq \frac{1}{\phi<sup>n-1</sup>}[/itex]mod(u1-\phi) and then conclude un converges to \phi


I have tried with the identity \phi = 1+ \frac{1}{\phi} if anything came to light... And I tried dividing the mods but it got even more complicated.

I can prove from the seocnd equation that un converges to \phi as n-1 converges to infinity and thus 1/+inf =0, the right side becomes zero and we get mod(un - \phi) \leq0 which is the definiton of convergence. But I can't get from the first equation to the second. I don't know how to pass from un-1 to u1 and the part of the \phin-1. Can someone shed some light on this issue?
 
Physics news on Phys.org
Calabi_Yau said:
Let Fn denote the Fibonacci sequence.
un is the sequence given by: un= Fn+1/Fn. Show that mod(un - \phi) \leq\frac{1}{\phi}mod(un-1-\phi) and therefore mod(un - \phi) \leq \frac{1}{\phi<sup>n-1</sup>}[/itex]mod(u1-\phi) and then conclude un converges to \phi


I have tried with the identity \phi = 1+ \frac{1}{\phi} if anything came to light... And I tried dividing the mods but it got even more complicated.

I can prove from the seocnd equation that un converges to \phi as n-1 converges to infinity and thus 1/+inf =0, the right side becomes zero and we get mod(un - \phi) \leq0 which is the definiton of convergence. But I can't get from the first equation to the second. I don't know how to pass from un-1 to u1 and the part of the \phin-1. Can someone shed some light on this issue?

We have
<br /> |u_n - \phi| \leq \frac{1}{\phi} |u_{n-1} - \phi| <br />
But since this is true for all n, we also have
<br /> |u_n - \phi| \leq \frac{1}{\phi} |u_{n-1} - \phi| \leq \frac{1}{\phi} \frac{1}{\phi} |u_{n-2} - \phi| = \frac{1}{\phi^2} |u_{n-2} - \phi|<br />
and we can clearly keep going, picking up a factor of \phi^{-1} each time, until we have a multiple of |u_1 - \phi| on the right.
 
Hmm, I didn't see that pattern. That is the same as making n-1=p and then doing the same for up and up-1.

Nice, thank you :)
 
Question: A clock's minute hand has length 4 and its hour hand has length 3. What is the distance between the tips at the moment when it is increasing most rapidly?(Putnam Exam Question) Answer: Making assumption that both the hands moves at constant angular velocities, the answer is ## \sqrt{7} .## But don't you think this assumption is somewhat doubtful and wrong?

Similar threads

  • · Replies 11 ·
Replies
11
Views
2K
  • · Replies 1 ·
Replies
1
Views
2K
  • · Replies 2 ·
Replies
2
Views
2K
  • · Replies 4 ·
Replies
4
Views
2K
  • · Replies 4 ·
Replies
4
Views
2K
Replies
6
Views
3K
  • · Replies 9 ·
Replies
9
Views
2K
  • · Replies 6 ·
Replies
6
Views
2K
  • · Replies 3 ·
Replies
3
Views
2K
  • · Replies 6 ·
Replies
6
Views
2K